- #1
- 3
- 1
Hello!
This is my first post on this forum, so make sure to tell me if I am doing something wrong :)
I was trying to derive the Lorentz factor today, and I used the following page as a guide. The top answer by Jimmy360 is what I followed.
https://physics.stackexchange.com/questions/173268/deriving-the-lorentz-factor-gamma
I was unable to solve for t' as they did, and it occurred to me that there was perhaps an issue with the derivation. Now I am in no way knowledgeable nor talented in derivations so I wanted to have someone look at my thought process. I have a hard time imagining that the derivation I linked is wrong, but then again I made it work for me with a small change that seems to make sense.
I realize that I am most likely wrong but I can't figure out why. That's why I need your help. If I'm misunderstanding something I want to find what that something is.
So, it seems that Jimmy360 set t' to be the coordinate time, and t to be the proper time.
Hence l=ct (that's an observation made by an observer in the reference frame of the rods), and d=ct' (that's an observation made by an observer in the other reference frame, which doesn't follow the rods)
Now when they find that d2= l2+(vt)2, I would have expected to see d2= l2+(vt')2 instead. The distance that the rods travel in the x-axis is not measured at all by an observer in the rods' reference frame, is it? That's why I expected d to be expressed in terms of f' rather than f
After that change is made we get
c2t'2=c2t2+v2t'2
and then
c2t2=c2t'2-v2t'2
From there I am able to solve for t and in doing so find the Lorentz factor.
Is my thought process correct? If not, can you guide me towards what I don't understand?
Thank you very much for your time, I appreciate it :)
This is my first post on this forum, so make sure to tell me if I am doing something wrong :)
I was trying to derive the Lorentz factor today, and I used the following page as a guide. The top answer by Jimmy360 is what I followed.
https://physics.stackexchange.com/questions/173268/deriving-the-lorentz-factor-gamma
I was unable to solve for t' as they did, and it occurred to me that there was perhaps an issue with the derivation. Now I am in no way knowledgeable nor talented in derivations so I wanted to have someone look at my thought process. I have a hard time imagining that the derivation I linked is wrong, but then again I made it work for me with a small change that seems to make sense.
I realize that I am most likely wrong but I can't figure out why. That's why I need your help. If I'm misunderstanding something I want to find what that something is.
So, it seems that Jimmy360 set t' to be the coordinate time, and t to be the proper time.
Hence l=ct (that's an observation made by an observer in the reference frame of the rods), and d=ct' (that's an observation made by an observer in the other reference frame, which doesn't follow the rods)
Now when they find that d2= l2+(vt)2, I would have expected to see d2= l2+(vt')2 instead. The distance that the rods travel in the x-axis is not measured at all by an observer in the rods' reference frame, is it? That's why I expected d to be expressed in terms of f' rather than f
After that change is made we get
c2t'2=c2t2+v2t'2
and then
c2t2=c2t'2-v2t'2
From there I am able to solve for t and in doing so find the Lorentz factor.
Is my thought process correct? If not, can you guide me towards what I don't understand?
Thank you very much for your time, I appreciate it :)